Difference between revisions of "2016 AIME II Problems/Problem 15"

m (Solution)
Line 2: Line 2:
  
 
==Solution==
 
==Solution==
Replace <math>\sum x_ix_j</math> with <math>\frac12\left(\left(\sum x_i\right)^2-\sum x_i^2\right)</math> and the second equation becomes <math>\sum\frac{x_i^2}{1-a_i}=\frac{1}{215}</math>. Conveniently, <math>\sum 1-a_i=215</math> so we get <math>\left(\sum 1-a_i\right)\left(\sum\frac{x_i^2}{1-a_i}\right)=1=\left(\sum x_i\right)^2</math>. This is the equality case of Cauchy so <math>x_i=c(1-a_i)</math> for some constant <math>c</math>. Using <math>\sum x_i=1</math>, we find <math>c=\frac{1}{215}</math> and thus <math>x_2=\frac{3}{860}</math>.
+
Replace <math>\sum x_ix_j</math> with <math>\frac12\left(\left(\sum x_i\right)^2-\sum x_i^2\right)</math> and the second equation becomes <math>\sum\frac{x_i^2}{1-a_i}=\frac{1}{215}</math>. Conveniently, <math>\sum 1-a_i=215</math> so we get <math>\left(\sum 1-a_i\right)\left(\sum\frac{x_i^2}{1-a_i}\right)=1=\left(\sum x_i\right)^2</math>. This is the equality case of Cauchy so <math>x_i=c(1-a_i)</math> for some constant <math>c</math>. Using <math>\sum x_i=1</math>, we find <math>c=\frac{1}{215}</math> and thus <math>x_2=\frac{3}{860}</math>. Thus, the desired answer is <math>860+3=\boxed{863}</math>.

Revision as of 22:12, 17 March 2016

For $1 \leq i \leq 215$ let $a_i = \dfrac{1}{2^{i}}$ and $a_216 = \dfrac{1}{2^{215}}$. Let $x_1, x_2, ..., x_215$ be positive real numbers such that $\sum_{i=1}^{215} x_i=1$ and $\sum_{i \leq i < j \leq 216} x_ix_j = \dfrac{107}{215} + \sum_{i=1}^{216} \dfrac{a_i x_i^{2}}{2(1-a_i)}$. The maximum possible value of $x_2=\dfrac{m}{n}$, where $m$ and $n$ are relatively prime positive integers. Find $m+n$.

Solution

Replace $\sum x_ix_j$ with $\frac12\left(\left(\sum x_i\right)^2-\sum x_i^2\right)$ and the second equation becomes $\sum\frac{x_i^2}{1-a_i}=\frac{1}{215}$. Conveniently, $\sum 1-a_i=215$ so we get $\left(\sum 1-a_i\right)\left(\sum\frac{x_i^2}{1-a_i}\right)=1=\left(\sum x_i\right)^2$. This is the equality case of Cauchy so $x_i=c(1-a_i)$ for some constant $c$. Using $\sum x_i=1$, we find $c=\frac{1}{215}$ and thus $x_2=\frac{3}{860}$. Thus, the desired answer is $860+3=\boxed{863}$.